Autor Tema: Intento de prueba del UTF por inducción

0 Usuarios y 1 Visitante están viendo este tema.

29 Diciembre, 2018, 08:15 pm
Leído 12873 veces

simpleimpar

  • $$\Large \color{#5372a0}\pi\,\pi$$
  • Mensajes: 97
  • País: es
  • Karma: +0/-0
  • Sexo: Masculino
Hola a todos
Me parece que es posible aplicar el método de inducción para la demostración del teorema último de fermat. De hecho la nota escrita por Fermat al margen de su Aritmética de Diofánto deja entrever dicha posibilidad.
La idea consiste en que se admite como demostrado que la suma de dos cubos de enteros positivos es un número comprendido entre los cubos de dos enteros positivos consecutivos y por tanto la raíz cúbica de esa suma de cubos es un número irracional. Si se admite como hipótesis que la suma de dos potencias de exponente \( n \) natural, de dos enteros naturales está comprendida entre las potencias \( n \)-simas de dos enteros consecutivos, su raíz \( n \)-sima será irracional y también irracional su raíz \( n+1 \)-ésima. Entonces como se cumple la relación

 \( a^{n+1} + b^{n+1} = (a^n + b^n)(a+b)-ab(a^{n-1} + b^{n-1}) \)

 al tomar las raíces de índice \( n+1 \) de esta igualdad, el segundo miembro será un número irracional, porque el irracional \( (a^n +b^n)^{1/(n+1)} \) "contamina de irracionalidad" el número que resulta del desarrollo de ese segundo miembro, de modo que es también irracional el primer miembro, es decir, el número \( (a^{n+1} +b^{n+1})^{1/(n+1)} \) y en consecuencia, si \( a^n + b^n \) no es potencia n-sima de entero positivo, tampoco es \( a^{n+1} + b^{n+1} \) potencia \( n+1 \) de entero positivo. 
  
Ruego me disculpéis esta intromisión. Tengo a disposición del que lo desee un pdf de tres hojas con el desarrollo de este asunto en "[email protected]".
Saludos cordiales

31 Diciembre, 2018, 11:06 am
Respuesta #1

simpleimpar

  • $$\Large \color{#5372a0}\pi\,\pi$$
  • Mensajes: 97
  • País: es
  • Karma: +0/-0
  • Sexo: Masculino
Este es el archivo pdf

01 Enero, 2019, 11:35 am
Respuesta #2

Luis Fuentes

  • el_manco
  • Administrador
  • Mensajes: 56,038
  • País: es
  • Karma: +0/-0
Hola

 He separado en un nuevo hilo tu propuesta de demostración, para no mezclar su posible discusión con el tema donde originalmente habías ubicado tus mensajes.

 En cuanto a lo que explicas en el PDF, a vuelapluma y sin entrar en otros detalles, en la página 2 de:

\( (a^{n+1} + b^{n+1})^{1/(n+1)} =((a^n + b^n)(a+b)-ab(a^{n-1} + b^{n-1}))^{1/(n+1)} \)

 deduces que, como \( (a^n + b^n)^{1/(n+1)} \) es irracional se deduce que toda la expresión del segundo miembro es irracional. Eso no tiene porque ser cierto. Es decir en general que \( A^{q} \) sea irracional no significa que \( (A+B)^q \) sea irracional.

 Por ejemplo \( 13^{1/3} \) es irracional pero \( (13+14)^{1/3}=3 \) NO es irracional.

Saludos.

02 Enero, 2019, 12:04 pm
Respuesta #3

simpleimpar

  • $$\Large \color{#5372a0}\pi\,\pi$$
  • Mensajes: 97
  • País: es
  • Karma: +0/-0
  • Sexo: Masculino
Muchas gracias Luis.

Es cierto que si la suma de "equis" sumandos es una potencia de base y exponente naturales, su raíz de índice el exponente en cuestión es un entero, y por tanto no irracional, aunque las raíces de ese índice de todos o algunos de los sumandos sean números irracionales.

Es cierto que es erróneo afirmar que \( [(a^n+b^n)(a + b)-ab(a^{n -1}+b^{n -1})]^{n + 1} \) es irracional porque lo es \( (a^n+b^n)^{1/(n+1)} \). Te ruego aceptes mis disculpas por esta imperdonable metedura de pata.
Es cierto que para sostener esa afirmación hay que demostrar que si \( (a^n+b^n)^{1/(n+1)} \), con \( a, b \), y \( n>2  \)números naturales es irracional, entonces \( [(a^n+b^n)(a + b)-ab(a^{n-1}+b^{n-1})]^{1/(n+1)} \) también lo es.
Esto equivale a demostrar, con \( p \) y \( q \) naturales, que si \( p^{n+1}<a^n+b^n<(p+1)^{n+1} \) entonces \( q^{n+1}<a^{n+1}+b^{n+1}= (a^n+b^n)(a+b)-ab(a^{n-1}+b^{n-1})<(q+1)^{n+1} \).
O también que los números \( A=(a^n +b^n)(a+b) \) y \( B=-ab(a^{n-1}+b^{n-1}) \) no suman una potencia de exponen \( n+1  \) y base \( a^{n+1}+b^{n+1} \).

Estoy en ello tras comprobar, que, como no puede ser de otra manera, con números asequibles, se verifican esas relaciones. Esta es la típica tarea de encontrar una demostración general que dé cuenta de todos los casos particulares.
Saludos cordiales. 

Mensaje corregido desde la administración.

02 Enero, 2019, 12:17 pm
Respuesta #4

sugata

  • $$\Large \color{#9c57a6}\pi\,\pi\,\pi\,\pi\,\pi\,\pi$$
  • Mensajes: 3,617
  • País: es
  • Karma: +1/-0
  • Sexo: Masculino
Por favor, usa Látex. Es complicado seguir tus mensajes sin Látex.
A parte de ser las reglas del foro.

02 Enero, 2019, 01:11 pm
Respuesta #5

Luis Fuentes

  • el_manco
  • Administrador
  • Mensajes: 56,038
  • País: es
  • Karma: +0/-0
Hola

simpleimpar: como te indica sugata, recuerda leer y seguir  las reglas del mismo así como el tutorial del LaTeX para escribir las fórmulas matemáticas correctamente.

Por esta vez te hemos corregido el mensaje desde la administración.

Estoy en ello tras comprobar, que, como no puede ser de otra manera, con números asequibles, se verifican esas relaciones. Esta es la típica tarea de encontrar una demostración general que dé cuenta de todos los casos particulares.

Que con ejemplos concretos se cumplan las relaciones, no dice demasiado sobre si tu intento de demostración tiene visos o no de llegar a buen puerto. Ejemplos concretos también muestran que el Teorema de Fermat es cierto; lo difícil es demostrarlo con toda generalidad.

Saludos.

03 Enero, 2019, 05:24 pm
Respuesta #6

simpleimpar

  • $$\Large \color{#5372a0}\pi\,\pi$$
  • Mensajes: 97
  • País: es
  • Karma: +0/-0
  • Sexo: Masculino
Hola
La supuesta solución en enteros positivos de la ecuación \( m^n = a^n + b^n \) si existe, no tiene dos números iguales y supongo \( m>a>b \).
Se tienen las siguientes relaciones:
\( a^{n+1}+b^{n+1}= (a^n+b^n)(a+b)-ab(a^{n-1}+b^{n-1}) \) y \( a^{n+1}-b^{n+1}= (a^n-b^n)(a+b)-ab(a^{n-1}-b^{n-1}) \)
Despejando a+b de la primera y sustituyendo ab de la segunda se llega a la relación \( ab^n=0 \) de modo que es falso que existe solución en enteros  positivos para la ecuación de Fermat.
Se puede ver esto en el archivo pdf adjunto.
Espero no haber metido la "gamba" y comentarios
Saludos muy cordiales a todos. 

03 Enero, 2019, 05:37 pm
Respuesta #7

Luis Fuentes

  • el_manco
  • Administrador
  • Mensajes: 56,038
  • País: es
  • Karma: +0/-0
Hola

La supuesta solución en enteros positivos de la ecuación \( m^n = a^n + b^n \) si existe, no tiene dos números iguales y supongo \( m>a>b \).
Se tienen las siguientes relaciones:
\( a^{n+1}+b^{n+1}= (a^n+b^n)(a+b)-ab(a^{n-1}+b^{n-1}) \) y \( a^{n+1}-b^{n+1}= (a^n-b^n)(a+b)-ab(a^{n-1}-b^{n-1}) \)
Despejando a+b de la primera y sustituyendo ab de la segunda se llega a la relación \( ab^n=0 \) de modo que es falso que existe solución en enteros  positivos para la ecuación de Fermat.

Dicho con todos los respetos, el argumento es muy ingenuo. No puedes pretender que de dos identidades que son ciertas para cualesquiera \( a,b \) (enteros, racionales reales) se llegue a que necesariamente \( a \) o \( b \) son nulos.

Si uno llega a algo así, lo único que puede hacer es revisar con mucho cuidado donde está el error en las cuentas.

Citar
Se puede ver esto en el archivo pdf adjunto.
Espero no haber metido la "gamba" y comentarios

Cuando despejas \( ab \) en la segunda ecuación el denominador que pones está mal.

Saludos.

03 Enero, 2019, 08:49 pm
Respuesta #8

simpleimpar

  • $$\Large \color{#5372a0}\pi\,\pi$$
  • Mensajes: 97
  • País: es
  • Karma: +0/-0
  • Sexo: Masculino
Tiene toda la razón Luis y como me temía he vuelto a errar. De nuevo ruego que aceptéis mis disculpas

23 Enero, 2019, 06:32 pm
Respuesta #9

simpleimpar

  • $$\Large \color{#5372a0}\pi\,\pi$$
  • Mensajes: 97
  • País: es
  • Karma: +0/-0
  • Sexo: Masculino
Hola a todos
Por si acaso sirve de algo os envío un pdf con una reflexión sobre el asunto
Saludos